- PowerScore Staff
- Posts: 5972
- Joined: Mar 25, 2011
- Fri Nov 03, 2017 1:31 pm
#41147
Complete Question Explanation
(The complete setup for this game can be found here: lsat/viewtopic.php?t=15621)
The correct answer choice is (A)
J must have been hired sixth, leaving six variables—H, F and I, D, E, B—hired before J. Consequently, the correct answer is six. As you might suspect, a number of students incorrectly selected answer choice (B) because they counted just the days before J. The question stem is specific in asking for the number of workers hired before J, not the number of days before J’s hiring. Remember—always read each question stem very closely!
(The complete setup for this game can be found here: lsat/viewtopic.php?t=15621)
The correct answer choice is (A)
J must have been hired sixth, leaving six variables—H, F and I, D, E, B—hired before J. Consequently, the correct answer is six. As you might suspect, a number of students incorrectly selected answer choice (B) because they counted just the days before J. The question stem is specific in asking for the number of workers hired before J, not the number of days before J’s hiring. Remember—always read each question stem very closely!
Dave Killoran
PowerScore Test Preparation
Follow me on X/Twitter at http://twitter.com/DaveKilloran
My LSAT Articles: http://blog.powerscore.com/lsat/author/dave-killoran
PowerScore Podcast: http://www.powerscore.com/lsat/podcast/
PowerScore Test Preparation
Follow me on X/Twitter at http://twitter.com/DaveKilloran
My LSAT Articles: http://blog.powerscore.com/lsat/author/dave-killoran
PowerScore Podcast: http://www.powerscore.com/lsat/podcast/